¿El operador hermitiano H=−d2dx2H=−d2dx2H=-\frac{d^2}{dx^2} tiene valores propios imaginarios?

En la mecánica cuántica, los operadores hermitianos juegan un papel muy importante porque poseen valores propios reales.

Considerando d 2 d X 2 , es un operador hermitiano (en realidad es el hamiltoniano más simple) y su función propia se puede expresar mediante mi i k X .

Sin embargo, ¿y si k es imaginario? si k es imaginario, d 2 d X 2 mi i k X = k 2 mi i k X todavía se mantiene, con su valor propio k 2 ¡imaginario!

¿Alguien puede decirme qué está mal aquí?

EDITAR: Como dice un comentario, el problema puede estar en la restricción de L 2 . Desafortunadamente, la mayoría de los libros de texto de física simplemente ignoran la restricción de L 2 . ¿Alguien puede darme una prueba rigurosa de que la onda simple mi i k X (con k real) es de alguna manera apropiado en la mayoría de las situaciones?

Relacionado: physics.stackexchange.com/q/81041/2451 y enlaces allí.
No, mi i k X no son funciones propias, no se encuentran en L 2 . Desde d 2 / d X 2 es simétrico/hermitiano, el espectro tiene que ser real.
Sí, sé que el problema puede estar en la restricción de L 2 . Desafortunadamente, la mayoría de los libros de texto de física simplemente ignoran la restricción de L 2 . ¿Alguien puede darme una prueba rigurosa de que la onda simple mi i k X es de alguna manera suficiente en la mayoría de las situaciones?
@Martin: cuidado con la terminología: los operadores hermitianos , para los cuales los dominios del operador y su adjunto no coinciden, tienen valores propios reales , pero pueden tener elementos imaginarios en sus espectros . Solo para los operadores autoadjuntos el espectro también es real, y solo para ellos tenemos una base de vectores propios reales de todo el espacio.
@ACuriousMind: Correcto. Lo sé, pero no debería haber sido indulgente ya que esta pregunta surge de un uso demasiado indulgente de las matemáticas en primer lugar...

Respuestas (2)

Sus "valores propios imaginarios" no funcionan, porque las funciones propias no son funciones propias. no mienten en L 2 , como usted parece ser consciente de.

Entonces, tratemos con el Laplaciano mismo: Δ = d 2 d X 2 . Lo que quiero hacer es calcular la transformada de Fourier de este operador, porque la transformada de Fourier diagonaliza Δ , como veremos. De la forma diagonal, podemos leer el espectro y así concluir que el espectro consiste en R + . Por supuesto, no respondo a la pregunta "¿por qué mi i k X son suficientes de verdad k ", simplemente porque es una pregunta sin sentido en el escenario del espacio de Hilbert.

Hagamoslo. tomar una función ψ L 2 ( R ) y calcula:

F ( Δ ψ ) ( k ) = mi i k X ( d 2 d X 2 ) ψ ( X ) d X = k 2 mi i k X ψ ( X ) d X = k 2 F ( ψ ) ( k )

donde integramos por partes dos veces (usando eso ψ necesariamente se desvanece en el infinito) y luego se diferencia mi i k X . Esta fórmula significa que F diagonaliza el laplaciano, porque acabamos de ver que la transformada de Fourier del laplaciano F ( Δ ) F es un operador de multiplicación. La idea ahora es que a partir de un operador de multiplicación, podemos leer el espectro: es el rango esencial del operador de multiplicación. Solo a partir de la definición, podemos ver que esto será [ 0 , ) en nuestro caso, por lo tanto σ ( Δ ) = [ 0 , ) .

En ningún momento hablamos de mi i k X , entonces, ¿cómo resolver el mi i k X con imaginario k ¿negocio? No tiene que hacerlo: ninguna de estas funciones está en L 2 , pero: si ahora pones ψ ( X ) = mi i k X con verdadero k (porque k es real en la transformada de Fourier!), entonces parece que es una función propia de Δ . En otras palabras: si imaginas un espacio cuya base son las funciones mi i k X , entonces la transformada de Fourier del Laplaciano es simplemente la matriz infinita con valores propios k 2 en la diagonal Sin embargo, ¡esta no es la imagen rigurosa!

Editar: aquí se puede encontrar una muy buena explicación con más matemáticas: https://math.stackexchange.com/questions/766479/what-is-spectrum-for-laplacian-in-mathbbrn Tenga en cuenta que esto no es matemática fácil, pero para entender los puntos más finos de este negocio, no puedes evitarlo.

Creo que es una buena explicación y un buen complemento para mi intento.

En primer lugar, si k =: i k es imaginario, el valor propio ("energía") es k 2 , es decir, real pero negativo:

d 2 d X 2 mi i k X = d 2 d X 2 mi k X = k 2 mi k X .

Físicamente, esa es una onda evanescente en una dirección, pero crece sin límites en la otra, así que si su espacio es todo X R , no es una función de onda válida por razones físicas muy tangibles: no es normalizable. La razón subyacente es que su energía es más pequeña que el mínimo del potencial en el que vive. Cualquier conferencia o libro introductorio de QM debe cubrir esto, y también presentar este tipo de función de onda (probablemente en el contexto de barreras potenciales finitas).

Para el lado más formal y matemático de las cosas, se debe recomendar este documento: http://arxiv.org/abs/quant-ph/9907069 .

En cuanto a las ondas planas, tampoco son estados realmente válidos (físicamente, una partícula nunca tendrá un momento perfectamente definido), pero son convenientes para trabajar y, a menudo, adecuadas (situación prototípica: un haz de partículas que incide sobre un objetivo descrita por una onda plana). Cuando una sola onda plana no es lo suficientemente buena (para ser más realistas, una partícula podría describirse como un paquete de ondas), aún puede hacer una descomposición en términos de ondas planas, es decir, una transformada de Fourier. Su estado real (buen comportamiento, matemáticamente válido) puede describirse como una suma sobre ondas planas.